Dudas varias años 2015, 2016, 2017, 2018

Foro de discusion Sobre RFH

Moderador: Alberto

Nodeno
Be
Mensajes: 34
Registrado: 06 Oct 2019, 17:08

Dudas varias años 2015, 2016, 2017, 2018

Mensaje por Nodeno »

Buenas noches a tod@s, haciendo exámenes de otros años me he encontrado con ciertos problemas que no he sido capaz de resolver, concretamente de los exámenes oficiales de los años 2015, 2016, 2017 y 2018. He buscado por el foro todos estos problemas, pero en muchos de ellos, no sé por qué motivo, pero no me deja ver las fórmulas que dejan los compañeros, y otros tantos no los he encontrado. Son bastantes problemas, agradecería muchísimo si alguien supiera y quisiera compartir su solución, y así poder ayudar a otras personas que se están preparando la prueba. Sin más dilatación (la térmica no) :lol: , allá van los problemas.

2015:

99. Se tiene el campo eléctrico producido por una
esfera uniformemente cargada con densidad de
carga eléctrica ρ. La carga total de la esfera es
Q y su radio es a. Φ es el flujo de dicho campo
eléctrico en una superficie esférica de radio b.
La distancia entre los centros de ambas esferas
es a+b. ¿Cuánto vale Φ?:
1. 0.
2. Q.
3. Q/ε0.
4. (b/a)×(Q/ε0).

Ni idea de como plantearla.

102. Se tiene un dipolo magnético m en el seno de
un campo magnético uniforme B. ¿Qué fuerza
ejerce el campo magnético sobre el dipolo?:
1. Nula.
2. Proporcional al producto escalar de los vectores
m y B.
3. Proporcional al producto vectorial de los
vectores m y B.
4. mB/2.

En esta he pensado que como U=-m*B, y F=-grad(U)=0, dado que U=cte. Es cierto esto?

115. Un condensador de capacidad C se encuentra
situado en serie con una resistencia de valor R.
El condensador se encuentra inicialmente cargado
cuando un interruptor se cierra en el
instante t = 0 para cerrar el circuito. ¿En qué
momento t ha disipado la resistencia la mitad
de la energía almacenada originalmente en el
condensador?:
1. R•C.
2. R•C•ln 2.
3. (R•C•ln 2) /2.
4. (R•C) /2.

Creo que se hace con las ecuaciones de carga y descarga del condensador, pero no me sale....

119. Un sistema está compuesto por una bola de
radio R cargada uniformemente y un medio
circundante de densidad de carga ρ=α/r, donde
α es una constante y r es la distancia desde el
centro de la bola hasta el punto analizado.
Hallar el valor de la carga de la bola para el
cual la intensidad del campo fuera de la bola
no depende de r:
1. q = 􀰗
􀬸 αR2.
2. q = 􀰗
􀬶 αR2.
3. q = 􀟨αR2.
4. q = 2􀟨αR2.

Ni idea

136. Un neutrón de masa mn y velocidad vn choca
elásticamente con un núcleo de carbono de
masa mC que se encuentra en reposo. ¿Cuál es
la velocidad (módulo, dirección y sentido) final
del neutrón?
1. (mC-mn)vn / (mC+mn), en la misma dirección
y sentido contrario que vn.

2. (mC-mn)vn / (mC+mn), en la misma dirección
y sentido que vn.
3. (mC+mn)vn / (mC-mn), en la misma dirección y
sentido contrario que vn.
4. (mC-mn)vn / (mC+mn), en dirección perpendicular
a vn.

Supongo que hay que aplicar la conservación del momento lineal y la energía, pero no consigo llegar al resultado final.

159. Una partícula de masa m se encuentra en el
estado fundamental de un pozo de potencial
unidimensional. Hallar la energía de dicho
estado según la teoría de Schrödinger sabiendo
que en los bordes del pozo la función de
onda Ψ toma el mismo valor, que a su vez es la
mitad del que presenta en el medio del pozo:
1. E0 = 􀔰􀰮􀰗􀰮
􀬵􀬼􀯠􀯔􀰮 .

2. E0 = 􀔰􀰮􀰗􀰮
􀬵􀬶􀯠􀯔􀰮 .
3. E0 = 􀔰􀰮􀰗􀰮
􀬼􀯠􀯔􀰮 .
4. E0 = 􀔰􀰮􀰗􀰮
􀬶􀯠􀯔􀰮 .

No sé cómo usar la información de que en los bordes del pozo la función de onda toma el mismo valor, y que a su vez es la mitad del que presenta en el medio del pozo.

213. La función lógica F= (X´+XY´)Z´ + XY es
equivalente a:
(X´, Y´e Z´son los complementos de las variables
lógicas X,Y,Z)
1. XY+Z´.
2. XY+Z.
3. XZ+Y.
4. YZ+X.

Usando las leyes del álgebra de Boole no consigo llegar al resultado correcto.

226. A 100 kHz la ganancia de un filtro es 1 y la
respuesta en frecuencia decrece con una pendiente
de -20 dB/dec, ¿cuánto es la ganancia a
200 kHz?:
1. 0.01.
2. 0.1.
3. 0.5.
4. 0.7.

¿Cuál es la ecuación que se usa aquí?

2016

116. Un amplificador Klystron tiene una ganancia
de 20 dB y sus guías de onda de entrada y salida
son del mismo tipo. Si el campo eléctrico en
la guía de entrada tiene una amplitud máxima
de 30kV/m, la amplitud máxima del campo
eléctrico en la guía de salida será:
1. 60 kV/m.
2. 300 kV/m.
3. 600 kV/m.
4. 3000 kV/m.

Aquí tampoco sé cuál es la ecuación a usar.

155. Para fotones de 1 MeV en agua el coeficiente
de atenuación (μ) es 0.071 cm-1 y el coeficiente
de absorción energético (μen) es 0.031 cm-1.
¿Cuál es la energía media absorbida en agua
en cada interacción?:
1. 440 keV.
2. 511 keV.
3. 1 MeV.
4. 1.022 MeV.

Ni idea de como se hace esta

232. Un circuito digital contador de módulo 12,
tiene:
1. 12 flip-flops.
2. 6 flip-flops.
3. 4 flip-flops.
4. 3 flip-flops.

¿Alguien me podría recomendar explicar lo que es un flip flop o recomendar alguna bibliografía al respecto? O una ecuación que relacione el número del modúlo con el número de flip-flops. Gracias! :lol:

2017

158. La energía de enlace de un núcleo de 24Na
(Z=11) es:
1. 22.5 MeV.
2. 388 MeV.
3. 95 MeV.
4. 194 MeV.

Ni idea, hay que saberse el peso en umas con decimales de protones y neutrones?

171. En el bombardeo de protones en reposo con
haces de protones se pueden producir pares
protón-antiprotón (p + p → p + p + p + 􀢖􀴥). Calcular
la energía umbral para que la reacción
sea posible:
1. 5.6 GeV.
2. 7.6 GeV.
3. 9.6 GeV.
4. 11.6 GeV.

No sé cómo plantearla

2018

45. Un ecógrafo Doppler emite ultrasonidos de
frecuencia f que se propagan a velocidad vu y se
reflejan en el corazón que se mueve con velocidad
vc, siendo vc<<vu. El desplazamiento máximo
de frecuencia Δf que registra en ecógrafo es
aproximadamente:
1. ± 𝑨𝑐
𝑨𝑣
f.
2. ± 𝑨𝑣
𝑨𝑐
f.
3. ±2 𝑨𝑐
𝑨𝑣
f.

4. ±2 𝑨𝑣
𝑨𝑐
f.

Supongo que hay que hacerlo con el efecto Doppler, pero no sé cómo plantearlo

100. ¿Cuál será la ganancia de un tríodo de vacío de
6x10³ ohmios de impedancia y factor de amplificación
m=25 que está conectado en un circuito
de resistencia de carga R=4x104 ohmios?:
1. 5.
2. 21.7.
3. 25.
4. 28.75.

Ni idea

122. Encontrar el módulo del vector polarización en
un material dieléctrico, asumido homogéneo e
isótropo, con 𝛆r = 2.8 si el módulo de D = 3,0·10-7
C/m2:
1. 1.07·10-7 C/m2.
2. 1.93·10-7 C/m2.
3. 4.67·10-7 C/m2.
4. 8.4·10-7 C/m2.

Supongo que se usará: D=epsilon0*E + P. Obtenemos a partir del dato D que nos dan en el enunciado, el campo E, mediante D=epsilon0epsilonrE=epsilonE, y posteriormente despejamos de la primera ecuación el vector P, pero por algún motivo no me sale

127. El deuterio fue descubierto en 1932 por Urey
observando el corrimiento de las líneas espectrales
con respecto a las del hidrógeno ordinario.
¿Qué separación mínima habría que ser
capaz de observar en la longitud de la onda
para encontrar un desdoblamiento en la primera
línea de la serie de Balmer en una mezcla de
hidrógeno y deuterio?
Dato: 𝐑∞ = 1.1 x 107m-1
1. 0.7·10-10 m.
2. 1.8·10-10 m.
3. 2.5·10-10 m.
4. 3.1·10-10 m.

Ni idea de cómo plantearlo

129. Los fotones de las transiciones de rayos X entre
las capas L (n=2) y K (n=1) de un elemento de
número atómico Z:
1. Tienen mayor energía cuanto menor es Z.
2. Tienen mayor longitud de onda cuanto menor
es Z.

3. Tienen una energía que aumenta linealmente
con Z.
4. Tienen la misma energía para algunos elementos
de Z diferente.

No entiendo de dónde sale ese resultado. Cuál es la lógica de que la energía sea menor cuanto menor es z?

134. El electrón del átomo de hidrógeno en el nivel
5p3/2 puede hacer una transición dipolar eléctrica
al estado:
1. 5d3/2.
2. 3s1/2.
3. 3p3/2.
4. 4f3/2.

No entiendo bien las reglas de selección...aplicadas a ejercicios

137. Un haz de electrones que se mueve rectilíneamente
a velocidad no relativista es acelerado en
la dirección de su movimiento de forma que
radia una potencia P en la dirección que forma
30º con la dirección del haz.
La potencia que radiará a 60º del haz es:
1. P/√3.
2. P/√2.
3. 2P.
4. 3P.

Cómo cambia la potencia radiada en función del ángulo?

143. Un rayo gamma interacciona con un protón en
reposo y produce un pión 𝝅+ y un neutrón.
¿Cuál es la energía umbral de la radiación
gamma?:
Masa(𝝅+)=139.57MeV/c2. Masa (protón)=938.35
MeV/c2. Masa(neutrón)=939.56 MeV/c2.
1. 151 MeV.
2. 302 MeV.
3. 1089 MeV.
4. 2179 MeV.

Hay una formulita por ahí, pero no sé cómo aplicarla a este ejercicio.

154. Sea la desintegración A→B+𝛃, donde ambos
núcleos tiene número másico par. Si A tiene
espín-paridad 0+, ¿Cuál de los siguientes valores
de espín-paridad de B está prohibido?:
1. 0+.
2. 2+.
3. 2-.
4. 3-.

Ni idea

180. Un electrón está confiando en el estado fundamental
de un oscilador armónico unidimensional,
tal que √〈(𝒜 − 〈𝒜〉)𝟗〉 = 10 -10 m. Encontrar
la energía (en eV) necesaria para excitar el electrón
al primer estado excitado:
1. 3.8 eV.
2. 5.6 eV.
3. 7.9 eV.
4. 11.2 eV.

Supongo que hay que usar el teorema de incertidumbre de Heisenber, pero no me sale.

187. Estimar la energía mínima que un protón debe
tener para atravesar la barrera de Coulomb del
núcleo de un átomo estacionario de Cl (Z=17).
Datos: e=1.6·10-19 C; K0=8.98755x109 Nm2 C-2:
1. 0.25 MeV.
2. 0.51 MeV.
3. 1.22 MeV.
4. 4.4 MeV.

Imagino que hay que usar U=(kze^2)/(R), z=17 pero no sé que R usar. 1 amstrong?

203. Dada una temperatura de 300K, determinar la
probabilidad de que un estado de energía de
3kT por encima del nivel de Fermi esté ocupado
por un electrón:
1. 4.74%.
2. 4.98%.
3. 95%.
4. 95.26%.

Alguna idea?

206. Se tiene silicio intrínseco a T=300 K. La vida
media de los portadores es 𝛖=10 𝛍s. Dicho semiconductor
se ilumina con la luz de longitud de
onda 𝝂=1 𝛍m de forma que se absorbe uniformemente
una densidad de potencia óptica de 1
mW·cm-3. Si la eficiencia cuántica es de ƞ=1/2,
calcular el exceso de portadores en el semiconductor:
1. 3.01·1011 cm-3.
2. 1.01·1011 cm-3.
3. 5.07·1010 cm-3.
4. 2.53·1010cm-3.

No sé cómo usar el dato de la eficiencia cuántica

214. En un detector de centelleo ideal, si el fotopico
aparece para una energía E=4mec2, ¿dónde se
situará el borde Compton?:
1. 16/7 mec2.
2. 16/5 mec2.
3. 24/7 mec2.
4. 32/9 mec2.

Creo que hay una fórmula que relaciona la energía del fotopico con la energía del borde Compton, pero no la encuentro por ningún lado.

Muchas gracias por haber llegado hasta aquí por la paciencia. Agradezco toda ayuda. Muchas gracias! :lol: :lol: :lol:
simbiosiz
F
F
Mensajes: 82
Registrado: 20 Oct 2018, 14:03

Re: Dudas varias años 2015, 2016, 2017, 2018

Mensaje por simbiosiz »

Hola Nodeno, no sé si podré ayudarte con los problemas porque tengo poco tiempo para escribir por aquí, si en algún momento puedo lo intentaré.
Lo que sí puedo es decirte cómo ver las fórmulas en LaTeX que no se pueden ver en el foro. Aparecen como un enlace roto de una imagen, pero en realidad, en el propio enlace está la fórmula. Lo que yo hago es "click derecho" sobre el enlace -> "Abrir en pestaña nueva" o "Ver imagen", dependiendo del navegador que uses. Te aparecerá una web de la CIA, pero si te fijas en la barra de direcciones de ahí puedes copiar la parte de la ruta que contiene la fórmula.
Luego, en esta web:
https://www.codecogs.com/latex/eqneditor.php
pegas la fórmula y te sale.

Prefiero copiar la fórmula de la barra de direcciones porque si la copio directamente desde el menú que sale al hacer click derecho ahí me rellena los espacios con el caracter "%20" y tengo que andar borrándolo para que salga la fórmula correctamente.
Espero que te sirva.
Adjunto una imagen de ayuda

Un saludo
Adjuntos
1.png
1.png (271.46 KiB) Visto 12405 veces
simbiosiz
F
F
Mensajes: 82
Registrado: 20 Oct 2018, 14:03

Re: Dudas varias años 2015, 2016, 2017, 2018

Mensaje por simbiosiz »

2015:

99. Lo que plantean son dos esferas que se "besan", son tangentes pero ninguna contiene a la otra, entonces como la esfera de radio "b" no contiene ninguna carga en su interior, por la ley de Gauss el flujo a través de su superficie es nulo.

102. Un campo magnético nunca crea una fuerza neta sobre un dipolo magnético, lo que crea es un par de fuerzas (momento o torque o como lo quieras llamar). Hace que el dipolo gire para alinearse con el campo y ya está.

115. Recuerda que la energía almacenada en un condensador puede expresarse como:
U = ½ Q²/C
donde en este caso Q = Q₀·Exp[-t/RC]
con eso tienes todo lo necesario.

119. Este es un pelín más largo, solo te doy la pista. El campo eléctrico sólo depende de la carga dentro de la esfera de radio r minuscula. Y para hallar esa carga tienes que integrar.

136. Intenta de nuevo aplicar la conservación del momento lineal y la energía cinética considerando un choque frontal (simple, unidimensional) que te saldrá. A mi me ha salido.

159. Te tienes que repasar la teoría de los pozos de potencial cuántico. Esto te puede ayudar: http://www.sc.ehu.es/sbweb/fisica3/cuan ... /pozo.html
Ve a la sección de "pozo" no a la de "caja".

213. Voy a usar la numeración de aquí: https://image.slidesharecdn.com/leyesbo ... 1269403923
Ademas de las leyes de DeMorgan
Lo que está en el paréntesis: X'+XY' =>Ley 11=> X'+Y' =>DeMorgan=> (XY)'
Por tanto F = (XY)'Z'+XY =>Ley 11=> Z'+XY

226. Te piden V/V₀ y te dan decibelios. Usa
dB = 20 log₁₀(V/V₀) <=> V/V₀ = 10^(dB/20)
Usa escala logarítmica para el asunto de la pendiente. Como los decibelios decrecen a un ritmo de -20dB por década, los 200kHz pertenece a la década de 100kHz, entonces tienes:
-20dB · log₁₀(200kHz/100kHz) = -6,02dB
Y por tanto: V/V₀ = 10^(-6,02dB/20) ≈ 0,5

2016:

116. Quédate únicamente con lo de la ganancia, recuerda, lo que hemos aplicado en el ejercicio anterior para la ganancia en voltaje es análogo para la amplitud de los campos:
dB = 20 log₁₀(E/E₀) <=> E = E₀·10^(dB/20)
Que en nuestro caso es: E = E₀·10^(20dB/20) = E₀·10

155. E_absorbida/E_incidente = μ_en/μ => E_absorbida = E_incidente · μ_en/μ

232. Un Flip-Flop es un biestable. Un dispositivo digital capaz de almacenar 1 bit de información. Aquí no nos importa de qué tipo sea el biestable, lo que es importante es que cuando haces un contador con biestables cada biestable representa 1 bit. El módulo de un contador es el número máximo hasta el que cuenta, en este caso 12. Entonces ¿Cuántos bits necesitas para representar 12 en binario?
12₁₀ ≡ 1100₂ -> 4 bits. Pues necesitas 4 flip-flops (o biestables).

2017:

158. No es necesario hacer el típico cálculo en umas y tal. Recuerda que para la mayoría de núcleos la energía de enlace por nucleón es de unos 8 MeV.
http://hyperphysics.phy-astr.gsu.edu/hb ... /bcurv.gif
Por tanto, en este caso:
8 MeV · 24 = 192 MeV que se aproxima bastante al resultado.

171. Aquí tienes que usar esta fórmula que te será muy útil: E_umbral = -Q · Σ[m] / 2m_b
Donde:
Q = E_inicial - E_final = (m_iniciales - m_finales)·c²
Σ[m] es el sumatorio de todas las masas, iniciales + finales.
m_b es la masa del blanco, que en este caso es un protón.

---------
Veré si puedo con las de 2018 en otro momento.
Administrador
Site Admin
Mensajes: 437
Registrado: 16 Ago 2004, 22:27

Re: Dudas varias años 2015, 2016, 2017, 2018

Mensaje por Administrador »

Ya esta solucionado el problema de las formulas, ya se pueden ver y se pueden usar de nuevo, tan solo hay que añadirlas usando la etiqueta

Código: Seleccionar todo

[tex]x=\frac{-b\pm\sqrt{b^2-4ac}}{2a}[/tex]
\(x=\frac{-b\pm\sqrt{b^2-4ac}}{2a}\)
simbiosiz
F
F
Mensajes: 82
Registrado: 20 Oct 2018, 14:03

Re: Dudas varias años 2015, 2016, 2017, 2018

Mensaje por simbiosiz »

Administrador escribió: 02 Dic 2019, 09:44 Ya esta solucionado el problema de las formulas, ya se pueden ver y se pueden usar de nuevo, tan solo hay que añadirlas usando la etiqueta

Código: Seleccionar todo

[tex]x=\frac{-b\pm\sqrt{b^2-4ac}}{2a}[/tex]
\(x=\frac{-b\pm\sqrt{b^2-4ac}}{2a}\)
Muchas gracias Admin! :D
Nodeno
Be
Mensajes: 34
Registrado: 06 Oct 2019, 17:08

Re: Dudas varias años 2015, 2016, 2017, 2018

Mensaje por Nodeno »

Muchísimas gracias Simbiosiz por la respuesta tan rápida y completa! Me ha sido de gran utilidad! :D

Las he sabido hacer todas menos la 136 y la 159 del año 2015. No sé por qué me salen raíces y cosas raras en la que tengo que aplicar la conservación del momento lineal y la energía cinética, creo que me falta algún signo por poner. La del pozo no sé por dónde cogerla aún mirando la teoría. Estás hecho un máquina Simbiosiz ;)

Muchas gracias administrador por solucionar el tema de las fórmulas. Y a simbiosiz por redirigirnos a la página de la CIA para obtener el código a latex :bom:

Saludos
Nodeno
Be
Mensajes: 34
Registrado: 06 Oct 2019, 17:08

Re: Dudas varias años 2015, 2016, 2017, 2018

Mensaje por Nodeno »

Hola a todos.

He conseguido hacer los ejercicios 137, 143, 100, 122, 203 y 214 del examen de 2018.

Si alguien está interesado en saber su solución estaré encantado de dejarla aquí.

Saludos
simbiosiz
F
F
Mensajes: 82
Registrado: 20 Oct 2018, 14:03

Re: Dudas varias años 2015, 2016, 2017, 2018

Mensaje por simbiosiz »

Me alegro que vayas sacando los problemas Nodeno. Cuando los saca uno mismo ya es dificil que se escapen las próximas veces.
Voy a ayudarte con los del 2015 que se resisten. Y ya que tenemos LaTeX, me voy a atrever un poco.

2015

136.
Tenemos las dos ecuaciones de conservación en colisiones elásticas (momento lineal y energía cinética):
\(\left\{\begin{matrix}m_n v_n+m_c 0=m_n v+m_c v_c\\
m_n v_n^2+m_c 0^2=m_n v^2+m_c v_c^2\end{matrix}\right.\)
→ despejamos \(v_c\) de la primera expresión: \(v_c=(v_n-v)\frac{m_n}{m_c}\)

Y sustituimos en la segunda:

\(m_n v_n^2=m_n v^2+m_c\begin{bmatrix}(v_n-v)\frac{m_n}{m_c}\end{bmatrix}^2
=m_n v^2+(v_n^2+v^2-2vv_n)\frac{m_n^2}{m_c}\)


Agrupamos términos para que nos quede una ecuación de 2º grado en \(v\):

\(\begin{pmatrix}\frac{m_n^2}{m_c}+m_n\end{pmatrix}v^2-\frac{2m_n^2}{m_c}v_n\cdot v
+\begin{pmatrix}\frac{m_n^2}{m_c}-m_n\end{pmatrix}v_n^2=0\)


Y resolvemos:

\(\frac{\frac{2m_n^2}{m_c}v_n\pm \sqrt{\frac{4m_n^4}{m_c^2}v_n^2
-4\begin{pmatrix}\frac{m_n^2}{m_c}+m_n\end{pmatrix}
\begin{pmatrix}\frac{m_n^2}{m_c}-m_n\end{pmatrix}v_n^2}}
{2\begin{pmatrix}\frac{m_n^2}{m_c}+m_n\end{pmatrix}}=
\frac{\frac{2m_n^2}{m_c}v_n\pm \sqrt{\frac{4m_n^4}{m_c^2}v_n^2
-4\begin{pmatrix}\frac{m_n^4}{m_c^2}-m_n^2\end{pmatrix}v_n^2}}
{2\begin{pmatrix}\frac{m_n^2}{m_c}+m_n\end{pmatrix}}=\)


\(=\frac{\frac{2m_n^2}{m_c}v_n\pm \sqrt{4m_n^2v_n^2}}
{2\begin{pmatrix}\frac{m_n^2}{m_c}+m_n\end{pmatrix}}=

\frac{2\begin{pmatrix}\frac{m_n^2}{m_c}\pm m_n\end{pmatrix}v_n}
{2\begin{pmatrix}\frac{m_n^2}{m_c}+m_n\end{pmatrix}}=

\frac{\frac{m_c}{m_n}\begin{pmatrix}\frac{m_n^2}{m_c}\pm m_n\end{pmatrix}v_n}
{\frac{m_c}{m_n}\begin{pmatrix}\frac{m_n^2}{m_c}+m_n\end{pmatrix}}=

\frac{\begin{pmatrix}m_n\pm m_c\end{pmatrix}v_n}{m_n+m_c}\)


La solución con el "+" es \(v_n\), que es la situación inicial. La solución con el "−" nos da lo que nos piden.

------------------------------------------------------------------------

159.
Adjunto una imagen del pozo como ayuda. Nos piden el estado fundamental, es decir el nivel de abajo del todo. La solución general para la función de onda en un pozo es la que aparece en la página esta: http://www.sc.ehu.es/sbweb/fisica3/cuan ... /pozo.html

\(\Psi(x)=Ae^{ikx}+Be^{-ikx}\)
Con: \(k=\frac{\sqrt{2mE}}{\hbar}\)
Ellos lo llaman \(q\), yo lo llamo \(k\) porque me gusta más.

La situación que nos plantean es la de un pozo sencillo, totalmente simétrico respecto al centro del pozo. Por lo tanto, EN ESE CASO no tienes que andar complicandote la vida calculando los coeficientes A y B.
Como el estado fundamental tiene que ser una función par, \(\Psi(x)\) se puede expresar directamente como:

\(\Psi(x)=Ae^{ikx}+Be^{-ikx}\equiv C\cos{(kx)}\)

Y ahora sólo tenemos que aplicar las condiciones de contorno que nos dan para hallar k, que es de donde despejamos la energía E:

\(\cos{(ka)}=\frac{1}{2}\cos{(k0)}=\frac{1}{2}\cdot 1\;\Rightarrow\;ka=\frac{\pi}{3}\)

Entonces:

\(k=\frac{\sqrt{2mE}}{\hbar}=\frac{\pi}{3a}\;\Rightarrow\;E=\frac{1}{2m}\left(\frac{\hbar\pi}{3a}\right)^2=\frac{\hbar^2\pi^2}{18\,m\,a^2}\)

PD. Fíjate que \(a\) es la semi-anchura del pozo, es decir, el ancho del pozo es \(2a\), no lo mencionan en el enunciado pero es importante. Si nos hubieran dicho que el ancho del pozo es sólo \(a\) o cualquier otra cosa como \(\frac{5a}{6}\), el resultado sería un poco distinto, pero la forma de resolverlo sería la misma.
De hecho si nos diera la gana de que el ancho del pozo fuera:
  • \(\sqrt{\frac{8}{3}}\,a\) la solución correcta sería la 2.
  • \(\frac{4}{3}\,a\) la solución correcta sería la 3.
  • \(\frac{2}{3}\,a\) la solución correcta sería la 4.
Así que, poniéndonos estrictos, como el enunciado no especifica el ancho del pozo, todas las opciones son correctas, jajaja. Pero no vamos a ser tan malvados ¿no? :angel4:
Adjuntos
Finite-well-solutions.png
Finite-well-solutions.png (13.58 KiB) Visto 11844 veces
simbiosiz
F
F
Mensajes: 82
Registrado: 20 Oct 2018, 14:03

Re: Dudas varias años 2015, 2016, 2017, 2018

Mensaje por simbiosiz »

En la imagen del mensaje anterior el eje de abscisas tiene que ser de x, no de x/x0, y sin números, para que sea un pozo genérico, no uno concreto.
Lo subo de nuevo corregido porque no me deja editar el mensaje anterior.
Adjuntos
Finite-well-solutions.png
Finite-well-solutions.png (13.12 KiB) Visto 11839 veces
simbiosiz
F
F
Mensajes: 82
Registrado: 20 Oct 2018, 14:03

Re: Dudas varias años 2015, 2016, 2017, 2018

Mensaje por simbiosiz »

Vamos con los de 2018:

45.
El corazón realiza un movimiento de vaivén. Por tanto habrá 2 desplazamientos Doppler:

\(\left.\begin{matrix}f'_{va}=f\frac{v_u-v_c}{v_u}\\
f'_{ven}=f\frac{v_u+v_c}{v_u}\end{matrix}\right\}
\Delta f=f'_{ven}-f'_{va}=2\frac{v_c}{v_u}\)


Si tomamos el desplazamiento en el otro sentido:

\(\Delta f_{sentido\; contrario}=f'_{va}-f'_{ven}=-2\frac{v_c}{v_u}\)

De ahí: \(\Delta f=\pm 2\frac{v_c}{v_u}\)

--------------------
127.
Hay que usar la fórmula de Rydberg: \(\frac{1}{\lambda}=R_H\left (\frac{1}{n_f^2}-\frac{1}{n_i^2} \right )\)

Solo que para un isótopo hay que hacer una pequeña modificación a la constante de Rydberg. En general se cumple que:

\(\frac{R_{isotopo}}{\mu_{isotopo}}=\frac{R_H}{\mu_H}\) donde \(\mu\) es la masa reducida: \(\mu=\frac{m_{electron}\cdot M_{nucleo}}{m_{electron}+M_{nucleo}}\)

Entonces, para el deuterio: \(R_D=R_H\frac{M_D(m_e+M_H)}{M_H(m_e+M_D)}\)

Ya podemos calcular el desplazamiento en la longitud de onda, que para la primera línea de la serie de Balmer (\(n_f=2\) y \(n_i=3\)) es:

\(\Delta \lambda =\frac{1}{R_H\left (\frac{1}{n_f^2}-\frac{1}{n_i^2} \right )}-\frac{1}{R_D\left (\frac{1}{n_f^2}-\frac{1}{n_i^2} \right )}=
\frac{1}{R_H\left (\frac{1}{2^2}-\frac{1}{3^2} \right )}\left (1-\frac{M_H(m_e+M_D)}{M_D(m_e+M_H)}\right )=
\frac{36}{R_H\cdot 5}\left (1-\frac{1(m_e+M_D)}{2(m_e+M_H)}\right )\)


Si haces el cálculo te sale.

------------------------
129. Ley de Moseley.

------------------------
134.
Ahí están utilizando la siguiente notación: \(n\,l_j\)
Las reglas de selección para una transición DIPOLAR eléctrica son:
\(\left\{\begin{matrix}\begin{aligned}\Delta l&=\pm 1\\
\Delta j&=0,\pm 1,(0\rightarrow 0\text{ prohibida})\\
\Delta s&=0\end{aligned}\end{matrix}\right.\)

El número \(n\) no influye. El número \(s\) lo calculas a partir de \(j\) y \(l\). Recuerda que: \(j=l+s\)

Si haces los cálculos verás que la única opción que cumple todas las reglas es la 2.

------------------------
154.
Ahí has copiado mal el enunciado, es una desintegración alfa, no beta.
Mírate esto: http://nuclear.fis.ucm.es/FNYP-C/alfa-vijande.pdf
Páginas 12 y 13.

-------------------------
180.
Recuerda que en el oscilador armónico cuántico los niveles de energía están equiespaciados (Ver: Enlace):

\(\Delta E=E_{n+1}-E_n=\hbar\omega\)

El dato que te están dando es el valor esperado del desplazamiento, que en el oscilador armónico cuántico vale (Ver: Otro enlace):

\(\sqrt{\left \langle \left ( x-\left \langle x \right \rangle \right )^2 \right \rangle}\equiv \left \langle r \right \rangle=\sqrt{\frac{\hbar}{2m\omega }}\) → despejando omega: \(\omega =\frac{\hbar}{2m\left \langle r \right \rangle^2}\)

Lo introduces en \(\Delta E=\hbar\omega\) y listo.

---------------------------
187.
Has intuido bien, tienes que usar: \(U=k\frac{Z\,e^2}{R+r}\)
Donde:
  • \(r\) es el radio del protón: \(r\approx 1,25 \text{ fm}\)
  • \(R\) es el radio del núcleo, que para cualquier núcleo se calcula como: \(R\approx 1,25 \text{ fm}\cdot A^{1/3}\)
----------------------------
206.
Te dan:
  • Densidad de potencia óptica, que es la intensidad \(I\) que absorbe el cristal de silicio.
  • La vida media de los portadores \(\tau\).
  • La longitud de onda de la luz \(\lambda\), que te sirve para calcular la energía del fotón \(E_\gamma\).
  • La eficiencia cuántica misteriosa \(\eta\).
Pues el exceso de portadores será el número de portadores que no les da tiempo a recombinarse debido a la constante absorción de fotones. Si esa absorción se da con una eficiencia \(\eta\), el exceso de portadores será:

\(X=\eta \frac{I\, \tau }{E_\gamma }\)


Y creo que eso es todo por hoy, jeje.
simbiosiz
F
F
Mensajes: 82
Registrado: 20 Oct 2018, 14:03

Re: Dudas varias años 2015, 2016, 2017, 2018

Mensaje por simbiosiz »

En la 2018-180 el valor esperado del desplazamiento \(\left \langle r \right \rangle\) es la incertidumbre en la posición \(\Delta x\):
Ver: http://hyperphysics.phy-astr.gsu.edu/hb ... c4.html#c1
ogonzo_neutro
Li
Mensajes: 29
Registrado: 18 Abr 2019, 20:18

Re: Dudas varias años 2015, 2016, 2017, 2018

Mensaje por ogonzo_neutro »

simbiosiz escribió: 09 Dic 2019, 00:05 Y ahora sólo tenemos que aplicar las condiciones de contorno que nos dan para hallar k, que es de donde despejamos la energía E:

\(\cos{(ka)}=\frac{1}{2}\cos{(k0)}=\frac{1}{2}\cdot 1\;\Rightarrow\;ka=\frac{\pi}{3}\)

Entonces:

\(k=\frac{\sqrt{2mE}}{\hbar}=\frac{\pi}{3a}\;\Rightarrow\;E=\frac{1}{2m}\left(\frac{\hbar\pi}{3a}\right)^2=\frac{\hbar^2\pi^2}{18\,m\,a^2}\)

PD. Fíjate que \(a\) es la semi-anchura del pozo, es decir, el ancho del pozo es \(2a\), no lo mencionan en el enunciado pero es importante. Si nos hubieran dicho que el ancho del pozo es sólo \(a\) o cualquier otra cosa como \(\frac{5a}{6}\), el resultado sería un poco distinto, pero la forma de resolverlo sería la misma.
De hecho si nos diera la gana de que el ancho del pozo fuera:
  • \(\sqrt{\frac{8}{3}}\,a\) la solución correcta sería la 2.
  • \(\frac{4}{3}\,a\) la solución correcta sería la 3.
  • \(\frac{2}{3}\,a\) la solución correcta sería la 4.
Así que, poniéndonos estrictos, como el enunciado no especifica el ancho del pozo, todas las opciones son correctas, jajaja. Pero no vamos a ser tan malvados ¿no? :angel4:
Muy buenas

Muchísimas gracias, tanto a Nodeno por plantear las dudas, como especialmente a Simbiosiz por aclararlas.
Con respecto a este último, el 159 del examen de 2015, el hecho de que no especifiquen que 'a' es la semianchura del pozo no sería suficiente para anular la pregunta?
Casi me dirijo más bien a administradoras y residentes al respecto. No me parece serio que en un examen como este se dé por supuesto tal dato tan sólo porque es el método habitual de explicar este caso.
No sé si opinais lo mismo...

Un saludo y fuerzas! :)
Nodeno
Be
Mensajes: 34
Registrado: 06 Oct 2019, 17:08

Re: Dudas varias años 2015, 2016, 2017, 2018

Mensaje por Nodeno »

Madre mía! Vaya curradón de LaTeX te has pegado Simbiosiz! Muchísimas gracias por resolver todas las dudas con esa claridad y rapidez, me ha quedado todo muy claro. Muchísimas gracias!!!! Espero que ayude a toda la gente que lee el foro, seguro que sí. Por cierto simbiosiz vas a sacar la plaza fijo.

Respecto a lo que dices, gonzo, en mi opinión debería haberse anulado esa pregunta, pero viendo exámenes de otros años también te puedes encontrar auténticas barbaridades que han pasado por alto y no han anulado. En fin.

Un saludo a todos, y gracias de nuevo Simbiosiz.
ogonzo_neutro
Li
Mensajes: 29
Registrado: 18 Abr 2019, 20:18

Re: Dudas varias años 2015, 2016, 2017, 2018

Mensaje por ogonzo_neutro »

Hablando un poco de todo,
Cómo es que no está la plantilla de soluciones del examen de 2018 disponible en la página de la academia?
Y examen de este año?
simbiosiz
F
F
Mensajes: 82
Registrado: 20 Oct 2018, 14:03

Re: Dudas varias años 2015, 2016, 2017, 2018

Mensaje por simbiosiz »

Muchas gracias a los dos. Y gracias Nodeno por tus buenos ánimos, yo espero que todos los que lo estamos preparando tengamos nuestra plaza.

Respecto a lo de anular la pregunta, Ogonzo:
ogonzo_neutro escribió: 10 Dic 2019, 19:26 Con respecto a este último, el 159 del examen de 2015, el hecho de que no especifiquen que 'a' es la semianchura del pozo no sería suficiente para anular la pregunta?
Piensa que si has hecho bien la pregunta no vas a querer impugnarla. Por otro lado, para que la anulen, la comisión calificadora tiene que observar que hay razones de peso para ello y, en este caso, no creo que las haya. Como dice Nodeno, hay preguntas peores.
Una razón de peso sería por ejemplo que entre las opciones sí estuviera la solución para un ancho 'a' (como en la imagen, jajaj).
Adjuntos
Finite-well-solutions ancho'a'.png
Finite-well-solutions ancho'a'.png (17.75 KiB) Visto 11741 veces
Responder